Buscar

fundamentos de algebra 1

Faça como milhares de estudantes: teste grátis o Passei Direto

Esse e outros conteúdos desbloqueados

16 milhões de materiais de várias disciplinas

Impressão de materiais

Agora você pode testar o

Passei Direto grátis

Você também pode ser Premium ajudando estudantes

Faça como milhares de estudantes: teste grátis o Passei Direto

Esse e outros conteúdos desbloqueados

16 milhões de materiais de várias disciplinas

Impressão de materiais

Agora você pode testar o

Passei Direto grátis

Você também pode ser Premium ajudando estudantes

Faça como milhares de estudantes: teste grátis o Passei Direto

Esse e outros conteúdos desbloqueados

16 milhões de materiais de várias disciplinas

Impressão de materiais

Agora você pode testar o

Passei Direto grátis

Você também pode ser Premium ajudando estudantes
Você viu 3, do total de 4 páginas

Faça como milhares de estudantes: teste grátis o Passei Direto

Esse e outros conteúdos desbloqueados

16 milhões de materiais de várias disciplinas

Impressão de materiais

Agora você pode testar o

Passei Direto grátis

Você também pode ser Premium ajudando estudantes

Prévia do material em texto

18/06/2023, 17:24 Estácio: Alunos
https://simulado.estacio.br/bdq_simulados_avaliacao_parcial_resultado.asp?cod_hist_prova=311918023&cod_prova=6454624618&f_cod_disc= 1/4
 
Meus
Simulados
Teste seu conhecimento acumulado
Disc.: FUNDAMENTOS DE ÁLGEBRA   
Aluno(a): ISMAEL ANSELMO DA NOBREGA PEREIRA 202103124658
Acertos: 6,0 de 10,0 07/06/2023
Acerto: 0,0  / 1,0
O conjunto dos números reais e a operação multiplicação, possuem estrutura de grupo. Nestas condições, a
propriedade que garante que seja um grupo abeliano é:
Elemento neutro.
Elemento inverso.
 Associativa.
 Comutativa.
Distributiva.
Respondido em 07/06/2023 08:12:34
Acerto: 1,0  / 1,0
Marque a alternativa que indica a solução da equação 3x + 2 = 6x + 7 em Z8.
3
- 5/3
 1
2
4
Respondido em 07/06/2023 08:14:46
Acerto: 1,0  / 1,0
Seja (G,*) um grupo. Se R e S são subgrupos de G então R ∩ S é um subgrupo de G. Marque a alternativa
que apresenta a demonstração correta dessa proposição.
Por hipótese G é um grupo e R e S são subgrupos de G. R e S contém o
elemento e G, assim e R ∩ S  . Isso mostra que R ∩ S ≠ Ø.
Considere um elemento x R ∩ S  , temos x R e x S, pela
hipótese  x-1 R e x-1 S , temos então x-1  R ∩ S. Portanto, R
∩ S é um subgrupo de G.
∈ ∈
∈ ∈ ∈
∈ ∈ ∈
 Questão1
a
 Questão2
a
 Questão3
a
https://simulado.estacio.br/alunos/inicio.asp
javascript:voltar();
18/06/2023, 17:24 Estácio: Alunos
https://simulado.estacio.br/bdq_simulados_avaliacao_parcial_resultado.asp?cod_hist_prova=311918023&cod_prova=6454624618&f_cod_disc= 2/4
Por hipótese G é um grupo e R subgrupo de G. R contém o elemento e  
 G. Isso mostra que R ∩ S ≠ Ø. Considere dois elementos  x, y R
∩ S  .Pela hipótese xy R e xy S então  xy R ∩ S  . Agora
considerando um elemento x R ∩ S  , temos x R e x S,  pela
hipótese  x-1 R e x-1 S , temos então x-1 R ∩ S. Portanto, R
∩ S é um subgrupo de G.
Por hipótese G é um grupo e R e S são subgrupos de G. R e S contém o
elemento e  G, assim e R ∩ S . Isso mostra que R ∩ S ≠ Ø.
Considere dois elementos x, y R ∩ S. Pela teoria dos conjuntos x,y
 R e x,y S. Pela hipótese xy R e xy S então  xy R ∩ S  .
Portanto, R ∩ S é um subgrupo de G.
 Por hipótese G é um grupo e R e S são subgrupos de G. R e S contém o
elemento e G, assim e R ∩ S . Isso mostra que R ∩ S ≠ Ø.
Considere dois elementos x, y R ∩ S. Pela teoria dos conjuntos x,y
 R e x,y S. Pela hipótese  xy R e xy S então  xy R ∩ S  .
Agora considerando um elemento x R ∩ S  , temos x R e x S,
pela hipótese x-1 R e x-1 S , temos então x-1 R ∩ S.
Portanto, R ∩ S é um subgrupo de G.
Por hipótese G é um grupo e R e S são subgrupos de G. R e S contém o
elemento e G, assim e R ∩ S  . Considere dois elementos x, y
R ∩ S. Pela teoria dos conjuntos x,y R e x,y S. Agora
considerando um elemento x R ∩ S  , temos x R e x S, pela
hipótese x-1 R e x-1 S , temos então x-1 R ∩ S. Portanto, R ∩
S é um subgrupo de G.
Respondido em 07/06/2023 08:22:16
Acerto: 0,0  / 1,0
Considere  o  grupo multiplicativo G = {1, i, -1, -i}  e H = {1, -1} subgrupo de G. Marque a alternativa
que indica  as classes laterais G.
  {1, -1} , {i, - i}
{1, -1},  {i, - i}, {1, - i}
 {i, - i}
{1, -1},  {i, - i}, {i, -1}
{1, -1},  {i, - i}, {i, -1}, {-1, -1}
Respondido em 07/06/2023 08:25:52
Acerto: 0,0  / 1,0
∈ ∈
∈ ∈ ∈
∈ ∈ ∈
∈ ∈ ∈
∈ ∈
∈
∈ ∈ ∈ ∈ ∈
∈ ∈
∈
∈ ∈ ∈ ∈ ∈
∈ ∈ ∈
∈ ∈ ∈
∈ ∈
∈ ∈ ∈
∈ ∈ ∈
∈ ∈ ∈
 Questão4
a
 Questão5
a
18/06/2023, 17:24 Estácio: Alunos
https://simulado.estacio.br/bdq_simulados_avaliacao_parcial_resultado.asp?cod_hist_prova=311918023&cod_prova=6454624618&f_cod_disc= 3/4
N(f) = {2}.
N(f) = {1}.
 N(f) = {4}.
 N(f) = {0}
N(f) = {3}
Respondido em 07/06/2023 08:26:19
Acerto: 0,0  / 1,0
Considere as operações x * y = x + y - 2  e  x y = xy - 2x - 2y + a, com . Para que valor de a, (Z, * , ) é um
anel?
a = 1
a = 3
 a = - 2
 a = 6
a = 2
Respondido em 07/06/2023 08:29:48
Acerto: 1,0  / 1,0
Considere as seguintes a�rmações: 
(I) Se (A,+, .) é um anel comutativo, então (AK, +, .) é comutativo.  
(II) Se A e B são anéis com unidade, então A x B não tem unidade.
(III) Se (A,+, .) é um anel com unidade, então (Mnxn(A),+, .) tem unidade. 
(IV) (Zm , +, .) é um anel comutativo com unidade. 
 
Com relação as a�rmações podemos concluir que:
 
Somente a III e IV estão corretas.
Somente a II e III estão corretas.
Somente a I está correta.
Somente a II e IV estão corretas.
 Somente a I, III e IV estão corretas.
Respondido em 07/06/2023 08:31:21
Acerto: 1,0  / 1,0
O anel Z6 admite quantos divisores de zero?
1
4
Δ a ∈ Z Δ
 Questão6
a
 Questão7
a
 Questão8
a
18/06/2023, 17:24 Estácio: Alunos
https://simulado.estacio.br/bdq_simulados_avaliacao_parcial_resultado.asp?cod_hist_prova=311918023&cod_prova=6454624618&f_cod_disc= 4/4
5
 3
2
Respondido em 07/06/2023 08:33:10
Acerto: 1,0  / 1,0
Em Z4 = {0,1,2,3}, determine U(Z4) .
 U(Z4) = {1,3}
 U(Z4) = {0,1,3}
U(Z4) = {0,1,2}
U(Z4) = {2,3}
U(Z4) = {1,2,3}
Respondido em 07/06/2023 08:36:44
Acerto: 1,0  / 1,0
Marque a alternativa correta.
Considere um anel (Q, +, .) e I = Z (conjunto dos números pares). Z é um ideal no anel Q.
Seja I = {f: R → R/f(1) + f(2) = 0}  e (RR, +, .). I é um ideal do anel (RR, +, .).
O conjunto dos números pares não é um ideal principal de Z gerado pelo elemento 2.
 2Z é um ideal no anel Z.
Seja I é um ideal do anel A com unidade. Se I contém um elemento inversível de A, então I ≠ A.
Respondido em 07/06/2023 08:35:57
 Questão9
a
 Questão10
a

Continue navegando